Difference between revisions of "2002 AMC 12B Problems/Problem 9"
(→Solution) |
(→Solution) |
||
Line 9: | Line 9: | ||
==Solution== | ==Solution== | ||
− | We can let a=1, b=2, c=3, and d=4. <math>\frac{a}{d}=\frac{1}{4} | + | We can let a=1, b=2, c=3, and d=4. <math>\frac{a}{d}=\frac{1}{4} \Rightarrow \boxed{\mathrm{(C)}}</math> |
==See also== | ==See also== |
Revision as of 08:49, 5 February 2008
Problem
If are positive real numbers such that form an increasing arithmetic sequence and form a geometric sequence, then is
Solution
We can let a=1, b=2, c=3, and d=4.